area !!!!!!

Calculus Level 4

Given that g ( x ) g(x) is the inverse of f ( x ) = x 3 + 3 x + 1 f(x) = x^3+3x+1 , find the area enclosed by y = g ( x ) y=g(x) , x = 1 x=1 , x = 37 x=37 , and the x x -axis.


The answer is 74.25.

This section requires Javascript.
You are seeing this because something didn't load right. We suggest you, (a) try refreshing the page, (b) enabling javascript if it is disabled on your browser and, finally, (c) loading the non-javascript version of this page . We're sorry about the hassle.

3 solutions

I = 1 37 g ( x ) d x Let y = f ( x ) = x 3 + 3 x + 1 = 1 37 x d y Note: { y = 1 x = 0 y = 37 x = 3 = 0 3 x ( 3 x 2 + 3 ) d x and d y = ( 3 x 2 + 3 ) d x = 0 3 3 ( x 3 + x ) d x = 3 [ x 4 4 + x 2 2 ] 0 3 = 74.25 \begin{aligned} I & = \int_1^{37} g(x) \ dx & \small \color{#3D99F6} \text{Let }y = f(x) = x^3+3x+1 \\ & = \int_1^{37} x \ dy & \small \color{#3D99F6} \text{Note: }\begin{cases} y = 1 & \implies x = 0 \\ y =37 & \implies x=3 \end{cases} \\ & = \int_0^3 x \color{#3D99F6} \left(3x^2+3\right) dx & \small \color{#3D99F6} \text{and }dy = (3x^2 + 3) \ dx \\ & = \int_0^3 3\left(x^3+x\right) dx \\ & = 3 \left[\frac {x^4}4+\frac {x^2}2 \right]_0^3 \\ & = \boxed{74.25} \end{aligned}

@Chew-Seong Cheong Sir , shouldn't the limits be changed to (0,3) from (1,37) , after we switch the variable to x from y ie. step 3?

Ankit Kumar Jain - 2 years, 10 months ago

Log in to reply

Thanks, I have changed it.

Chew-Seong Cheong - 2 years, 10 months ago
Debarya Das
May 22, 2016

Since g ( x ) g\left( x \right) is inverse of f ( x ) f\left( x \right)

Therfore f ( x ) f\left( x \right) is a mirror image of g ( x ) g\left( x \right) to mirror y = x y=x

Let initially a graph contains x = 1 , x = 37 x=1,\quad x=37 lines and y = g ( x ) y=g(x) curve

After take the mirror images of all lines and curves about y = x y=x

The graph now contains y = 1 , y = 37 y=1,\quad y=37 lines and y = f ( x ) y=f(x) curve as shown

(Note:Since f'(x) is positive hence graph is always increasing and hence gives one x-coordinate for one y-coordinate)

By solving f ( x ) = 1 f(x)=1 & f ( x ) = 37 f(x)=37 seperately we get

x = 0 x=0 for f ( x ) = 1 f(x)=1 , and, x = 3 x=3 for f ( x ) = 37 f(x)=37

Let O(0,1), A(0,37) and B(3,37) be three points

Here area of graph f ( x ) f(x) between y = 1 y=1 & y = 37 y=37 with y-axis is our answer which is equal to:

a r e a ( O A B ) = a r e a ( r e c t a n g l e b y x = 0 , y = 0 , x = 3 & y = 37 ) + a r e a ( f ( x ) w i t h x a x i s f r o m x = 0 t o x = 3 ) area(OAB)=area(rectangle\quad by\quad x=0,y=0,x=3\quad \& \quad y=37)+area(f(x)\quad with\quad x-axis\quad from\quad x=0\quad to\quad x=3) - eq(i)

Therefore area of f(x) with x-axis from x=0 to x=3 is

0 3 f ( x ) d x \int _{ 0 }^{ 3 }{ f(x)dx }

3 4 4 + 3 ( 3 ) 2 2 + 3 \frac { { 3 }^{ 4 } }{ 4 } +\frac { 3{ (3) }^{ 2 } }{ 2 } +3

And area of rectangle ans in equation (i) is

37 X 3 37\quad X\quad 3

And then using equation (i) we can get our answer

Same solution. +1!. In this way, we can also generalise this for any arbitrary function.

Mayank Chaturvedi - 5 years ago
Hasan Kassim
Jun 29, 2015

\ast g ( x ) g(x) is the inverse of f ( x ) f(x) .

\ast f ( x ) = 3 x 2 + 3 > 0 f'(x) = 3x^2+3 >0 for all x x , so f ( x ) f(x) and g ( x ) g(x) are strictly increasing.(Symmetric about the line y = x y=x ).

f ( 0 ) = 1 \ast f(0) =1 then g ( 1 ) = 0 g(1) = 0

Therefore, g ( x ) 0 g(x) \geq 0 for x [ 1 , 37 ] x \in [1,37] Therefore our area is:

A = 1 37 g ( x ) d x \displaystyle A = \int_1^{37} g(x) dx

Let u = g ( x ) u= g(x) :

x = f ( u ) = > d x = f ( u ) d u \displaystyle x=f(u) => dx = f'(u) du

And since f ( 0 ) = 1 , f ( 3 ) = 37 f(0) =1 , f(3) = 37 , the integral becomes:

A = 0 3 u f ( u ) d u \displaystyle A= \int_0^3 uf'(u) du

= 0 3 u ( 3 u 2 + 3 u ) d u \displaystyle = \int_0^3 u(3u^2+3u) du

The rest is easy work that will lead to the final answer:

A = 74.25 \displaystyle \boxed{A= 74.25 }

Moderator note:

The approach that I prefer, would be to relate the integral of g, to the integral of f. This avoids having to find what f f' is, and relates to a wider class of functions.

If f f is a bijective function such that f ( 0 ) = 0 f(0) = 0 and f ( 1 ) = 1 f(1) =1 with inverse g g , then we have

0 1 f + g d x = 1 \int_0^1 f + g \, dx = 1

This can be seen by integrating f f with respect to x x , and then integrating g g with respect to y y . That is what you're essentially doing with "Let u = g ( x ) , d u = f ( u ) d u u = g(x), du = f'(u) \, du .

0 pending reports

×

Problem Loading...

Note Loading...

Set Loading...